edited by
910 views
4 votes
4 votes

edited by

1 Answer

Best answer
5 votes
5 votes

Let us consider L(M) first :

So going by the automata from start state to final state , we get  :  a*b*a*

Now we consider L(N) :

Collapsing to single state , we get two incoming loops on q0 ,

i.e. a and bb*  So we can write the regular expression as : (a + bb*)*

Now setting b* to ϵ , we get   :  (a + b)* 

Now obviously  L(M) is the subset of L(N).

Hence L(M)   ∩    L(N)   =   a*b*a*

Hence A) option should be correct .

selected by

Related questions

3 votes
3 votes
3 answers
1
Hirak asked May 22, 2019
1,388 views
How many $2$ state DFA’s with the designated initial state can be constructed over the alphabet over the alphabet $\sum = \{a, b\}$ that accept universal language?$4$$1...
0 votes
0 votes
3 answers
4